In a $\triangle ABC$, if $L$ and $M$ are points on $AB$ and $AC$ respectively such that $LM \| BC$.
Prove that $ar(\triangle LBC) = ar(\triangle MBC)$.


Given:

In a $\triangle ABC$, $L$ and $M$ are points on $AB$ and $AC$ respectively such that $LM \| BC$.

To do:

We have to prove that $ar(\triangle LBC) = ar(\triangle MBC)$.

Solution:

Join $LM, LC$ and $MB$.


$L$ and $M$ are the mid points of $AB$ and $AC$.

This implies,

$LM \| BC$

$\triangle LBM$ and $\triangle LCM$ are on the same base $LM$ and between the same parallels.

Therefore,

$ar(\triangle LBM) = ar(\triangle LCM)$......…(i)

$ar(\triangle LCM) = ar(\triangle LBM)

$\triangle LBC$ and $\triangle MBC$ are on the same base $BC$ and between the same parallels.

Therefore,

$ar(\triangle LBC) = ar(\triangle MBC)$......…(ii)

Hence proved. 

Tutorialspoint
Tutorialspoint

Simply Easy Learning

Updated on: 10-Oct-2022

32 Views

Kickstart Your Career

Get certified by completing the course

Get Started
Advertisements